Related rates, my answer differs from the book, misprint or me?How fast is the length of his shadow on the building decreasing when he is 4 m from the buildingRelated Rates: rate of change of the distance between two objectsRelated Rates change in theta

Single word request: Harming the benefactor

Does "Until when" sound natural for native speakers?

How is the wildcard * interpreted as a command?

Accountant/ lawyer will not return my call

When traveling to Europe from North America, do I need to purchase a different power strip?

Recommendation letter by significant other if you worked with them professionally?

Is it "Vierergruppe" or "Viergruppe", or is there a distinction?

'The literal of type int is out of range' con número enteros pequeños (2 dígitos)

Is it possible to avoid unpacking when merging Association?

Latex does not go to next line

In the quantum hamiltonian, why does kinetic energy turn into an operator while potential doesn't?

Why does Captain Marvel assume the people on this planet know this?

Why was Goose renamed from Chewie for the Captain Marvel film?

Reverse string, can I make it faster?

How to draw cubes in a 3 dimensional plane

How many characters using PHB rules does it take to be able to have access to any PHB spell at the start of an adventuring day?

Is it necessary to separate DC power cables and data cables?

Examples of a statistic that is not independent of sample's distribution?

Counting all the hearts

Are all players supposed to be able to see each others' character sheets?

How did Alan Turing break the enigma code using the hint given by the lady in the bar?

PTIJ: Should I kill my computer after installing software?

Why does liquid water form when we exhale on a mirror?

Word for a person who has no opinion about whether god exists



Related rates, my answer differs from the book, misprint or me?


How fast is the length of his shadow on the building decreasing when he is 4 m from the buildingRelated Rates: rate of change of the distance between two objectsRelated Rates change in theta













1












$begingroup$


Did my answer go wrong or does the book have a misprint?(there have been some inconstancies between the definitions used in the chapters and answer key, like two different authors, though only one is listed)



The problem: An airplane is flying 500 miles per hour horizontal one mile high over a radar station. Find the rate at which the distance is increasing when the plane is 2 miles from the station.



My answer is $1000/sqrt5$ or $200*sqrt5$



The book gives $250*sqrt3$



My method: triangle abc, a=1, b=2, and c is the hypotenuse, $db/dt=500$, and $c^2=1^2+b^2$, and I want $dc/dt$ at b=2



I took the derivative: $2c*fracdcdt=0+2b*fracdbdt$,



solved for $dc/dt$; $dc/dt=frac2b*db/dt2c$



Substitute the variables; $c=sqrt1+4$ and so $fracdcdt=frac2*2*5002*sqrt5=frac1000sqrt5$










share|cite|improve this question









New contributor




Max Power is a new contributor to this site. Take care in asking for clarification, commenting, and answering.
Check out our Code of Conduct.







$endgroup$
















    1












    $begingroup$


    Did my answer go wrong or does the book have a misprint?(there have been some inconstancies between the definitions used in the chapters and answer key, like two different authors, though only one is listed)



    The problem: An airplane is flying 500 miles per hour horizontal one mile high over a radar station. Find the rate at which the distance is increasing when the plane is 2 miles from the station.



    My answer is $1000/sqrt5$ or $200*sqrt5$



    The book gives $250*sqrt3$



    My method: triangle abc, a=1, b=2, and c is the hypotenuse, $db/dt=500$, and $c^2=1^2+b^2$, and I want $dc/dt$ at b=2



    I took the derivative: $2c*fracdcdt=0+2b*fracdbdt$,



    solved for $dc/dt$; $dc/dt=frac2b*db/dt2c$



    Substitute the variables; $c=sqrt1+4$ and so $fracdcdt=frac2*2*5002*sqrt5=frac1000sqrt5$










    share|cite|improve this question









    New contributor




    Max Power is a new contributor to this site. Take care in asking for clarification, commenting, and answering.
    Check out our Code of Conduct.







    $endgroup$














      1












      1








      1





      $begingroup$


      Did my answer go wrong or does the book have a misprint?(there have been some inconstancies between the definitions used in the chapters and answer key, like two different authors, though only one is listed)



      The problem: An airplane is flying 500 miles per hour horizontal one mile high over a radar station. Find the rate at which the distance is increasing when the plane is 2 miles from the station.



      My answer is $1000/sqrt5$ or $200*sqrt5$



      The book gives $250*sqrt3$



      My method: triangle abc, a=1, b=2, and c is the hypotenuse, $db/dt=500$, and $c^2=1^2+b^2$, and I want $dc/dt$ at b=2



      I took the derivative: $2c*fracdcdt=0+2b*fracdbdt$,



      solved for $dc/dt$; $dc/dt=frac2b*db/dt2c$



      Substitute the variables; $c=sqrt1+4$ and so $fracdcdt=frac2*2*5002*sqrt5=frac1000sqrt5$










      share|cite|improve this question









      New contributor




      Max Power is a new contributor to this site. Take care in asking for clarification, commenting, and answering.
      Check out our Code of Conduct.







      $endgroup$




      Did my answer go wrong or does the book have a misprint?(there have been some inconstancies between the definitions used in the chapters and answer key, like two different authors, though only one is listed)



      The problem: An airplane is flying 500 miles per hour horizontal one mile high over a radar station. Find the rate at which the distance is increasing when the plane is 2 miles from the station.



      My answer is $1000/sqrt5$ or $200*sqrt5$



      The book gives $250*sqrt3$



      My method: triangle abc, a=1, b=2, and c is the hypotenuse, $db/dt=500$, and $c^2=1^2+b^2$, and I want $dc/dt$ at b=2



      I took the derivative: $2c*fracdcdt=0+2b*fracdbdt$,



      solved for $dc/dt$; $dc/dt=frac2b*db/dt2c$



      Substitute the variables; $c=sqrt1+4$ and so $fracdcdt=frac2*2*5002*sqrt5=frac1000sqrt5$







      related-rates






      share|cite|improve this question









      New contributor




      Max Power is a new contributor to this site. Take care in asking for clarification, commenting, and answering.
      Check out our Code of Conduct.











      share|cite|improve this question









      New contributor




      Max Power is a new contributor to this site. Take care in asking for clarification, commenting, and answering.
      Check out our Code of Conduct.









      share|cite|improve this question




      share|cite|improve this question








      edited 2 days ago







      Max Power













      New contributor




      Max Power is a new contributor to this site. Take care in asking for clarification, commenting, and answering.
      Check out our Code of Conduct.









      asked 2 days ago









      Max PowerMax Power

      1085




      1085




      New contributor




      Max Power is a new contributor to this site. Take care in asking for clarification, commenting, and answering.
      Check out our Code of Conduct.





      New contributor





      Max Power is a new contributor to this site. Take care in asking for clarification, commenting, and answering.
      Check out our Code of Conduct.






      Max Power is a new contributor to this site. Take care in asking for clarification, commenting, and answering.
      Check out our Code of Conduct.




















          2 Answers
          2






          active

          oldest

          votes


















          1












          $begingroup$

          It may be a misprint, but I don't think your answer is right either. The question asks for $dc/dt$ when the distance from the station is 2 - this means when c=2 not when b=2. Other than that your answer is correct; plugging a=1, b=$sqrt 3$, c=2 in, I get $250sqrt3$ (not $250/sqrt3$)






          share|cite|improve this answer








          New contributor




          Ben is a new contributor to this site. Take care in asking for clarification, commenting, and answering.
          Check out our Code of Conduct.






          $endgroup$








          • 1




            $begingroup$
            Ah semantics. Probably not the last time they get the better of me. And yes $250/sqrt3$, was a typo on my part, fixed now.
            $endgroup$
            – Max Power
            2 days ago



















          1












          $begingroup$

          You approach is good, you just mixed up $b$ and $c$. When plane is two miles from the station, $c=2$, $b=sqrt3$. So I think the answer should be $250sqrt3$.






          share|cite|improve this answer









          $endgroup$












            Your Answer





            StackExchange.ifUsing("editor", function ()
            return StackExchange.using("mathjaxEditing", function ()
            StackExchange.MarkdownEditor.creationCallbacks.add(function (editor, postfix)
            StackExchange.mathjaxEditing.prepareWmdForMathJax(editor, postfix, [["$", "$"], ["\\(","\\)"]]);
            );
            );
            , "mathjax-editing");

            StackExchange.ready(function()
            var channelOptions =
            tags: "".split(" "),
            id: "69"
            ;
            initTagRenderer("".split(" "), "".split(" "), channelOptions);

            StackExchange.using("externalEditor", function()
            // Have to fire editor after snippets, if snippets enabled
            if (StackExchange.settings.snippets.snippetsEnabled)
            StackExchange.using("snippets", function()
            createEditor();
            );

            else
            createEditor();

            );

            function createEditor()
            StackExchange.prepareEditor(
            heartbeatType: 'answer',
            autoActivateHeartbeat: false,
            convertImagesToLinks: true,
            noModals: true,
            showLowRepImageUploadWarning: true,
            reputationToPostImages: 10,
            bindNavPrevention: true,
            postfix: "",
            imageUploader:
            brandingHtml: "Powered by u003ca class="icon-imgur-white" href="https://imgur.com/"u003eu003c/au003e",
            contentPolicyHtml: "User contributions licensed under u003ca href="https://creativecommons.org/licenses/by-sa/3.0/"u003ecc by-sa 3.0 with attribution requiredu003c/au003e u003ca href="https://stackoverflow.com/legal/content-policy"u003e(content policy)u003c/au003e",
            allowUrls: true
            ,
            noCode: true, onDemand: true,
            discardSelector: ".discard-answer"
            ,immediatelyShowMarkdownHelp:true
            );



            );






            Max Power is a new contributor. Be nice, and check out our Code of Conduct.









            draft saved

            draft discarded


















            StackExchange.ready(
            function ()
            StackExchange.openid.initPostLogin('.new-post-login', 'https%3a%2f%2fmath.stackexchange.com%2fquestions%2f3141677%2frelated-rates-my-answer-differs-from-the-book-misprint-or-me%23new-answer', 'question_page');

            );

            Post as a guest















            Required, but never shown

























            2 Answers
            2






            active

            oldest

            votes








            2 Answers
            2






            active

            oldest

            votes









            active

            oldest

            votes






            active

            oldest

            votes









            1












            $begingroup$

            It may be a misprint, but I don't think your answer is right either. The question asks for $dc/dt$ when the distance from the station is 2 - this means when c=2 not when b=2. Other than that your answer is correct; plugging a=1, b=$sqrt 3$, c=2 in, I get $250sqrt3$ (not $250/sqrt3$)






            share|cite|improve this answer








            New contributor




            Ben is a new contributor to this site. Take care in asking for clarification, commenting, and answering.
            Check out our Code of Conduct.






            $endgroup$








            • 1




              $begingroup$
              Ah semantics. Probably not the last time they get the better of me. And yes $250/sqrt3$, was a typo on my part, fixed now.
              $endgroup$
              – Max Power
              2 days ago
















            1












            $begingroup$

            It may be a misprint, but I don't think your answer is right either. The question asks for $dc/dt$ when the distance from the station is 2 - this means when c=2 not when b=2. Other than that your answer is correct; plugging a=1, b=$sqrt 3$, c=2 in, I get $250sqrt3$ (not $250/sqrt3$)






            share|cite|improve this answer








            New contributor




            Ben is a new contributor to this site. Take care in asking for clarification, commenting, and answering.
            Check out our Code of Conduct.






            $endgroup$








            • 1




              $begingroup$
              Ah semantics. Probably not the last time they get the better of me. And yes $250/sqrt3$, was a typo on my part, fixed now.
              $endgroup$
              – Max Power
              2 days ago














            1












            1








            1





            $begingroup$

            It may be a misprint, but I don't think your answer is right either. The question asks for $dc/dt$ when the distance from the station is 2 - this means when c=2 not when b=2. Other than that your answer is correct; plugging a=1, b=$sqrt 3$, c=2 in, I get $250sqrt3$ (not $250/sqrt3$)






            share|cite|improve this answer








            New contributor




            Ben is a new contributor to this site. Take care in asking for clarification, commenting, and answering.
            Check out our Code of Conduct.






            $endgroup$



            It may be a misprint, but I don't think your answer is right either. The question asks for $dc/dt$ when the distance from the station is 2 - this means when c=2 not when b=2. Other than that your answer is correct; plugging a=1, b=$sqrt 3$, c=2 in, I get $250sqrt3$ (not $250/sqrt3$)







            share|cite|improve this answer








            New contributor




            Ben is a new contributor to this site. Take care in asking for clarification, commenting, and answering.
            Check out our Code of Conduct.









            share|cite|improve this answer



            share|cite|improve this answer






            New contributor




            Ben is a new contributor to this site. Take care in asking for clarification, commenting, and answering.
            Check out our Code of Conduct.









            answered 2 days ago









            Ben Ben

            263




            263




            New contributor




            Ben is a new contributor to this site. Take care in asking for clarification, commenting, and answering.
            Check out our Code of Conduct.





            New contributor





            Ben is a new contributor to this site. Take care in asking for clarification, commenting, and answering.
            Check out our Code of Conduct.






            Ben is a new contributor to this site. Take care in asking for clarification, commenting, and answering.
            Check out our Code of Conduct.







            • 1




              $begingroup$
              Ah semantics. Probably not the last time they get the better of me. And yes $250/sqrt3$, was a typo on my part, fixed now.
              $endgroup$
              – Max Power
              2 days ago













            • 1




              $begingroup$
              Ah semantics. Probably not the last time they get the better of me. And yes $250/sqrt3$, was a typo on my part, fixed now.
              $endgroup$
              – Max Power
              2 days ago








            1




            1




            $begingroup$
            Ah semantics. Probably not the last time they get the better of me. And yes $250/sqrt3$, was a typo on my part, fixed now.
            $endgroup$
            – Max Power
            2 days ago





            $begingroup$
            Ah semantics. Probably not the last time they get the better of me. And yes $250/sqrt3$, was a typo on my part, fixed now.
            $endgroup$
            – Max Power
            2 days ago












            1












            $begingroup$

            You approach is good, you just mixed up $b$ and $c$. When plane is two miles from the station, $c=2$, $b=sqrt3$. So I think the answer should be $250sqrt3$.






            share|cite|improve this answer









            $endgroup$

















              1












              $begingroup$

              You approach is good, you just mixed up $b$ and $c$. When plane is two miles from the station, $c=2$, $b=sqrt3$. So I think the answer should be $250sqrt3$.






              share|cite|improve this answer









              $endgroup$















                1












                1








                1





                $begingroup$

                You approach is good, you just mixed up $b$ and $c$. When plane is two miles from the station, $c=2$, $b=sqrt3$. So I think the answer should be $250sqrt3$.






                share|cite|improve this answer









                $endgroup$



                You approach is good, you just mixed up $b$ and $c$. When plane is two miles from the station, $c=2$, $b=sqrt3$. So I think the answer should be $250sqrt3$.







                share|cite|improve this answer












                share|cite|improve this answer



                share|cite|improve this answer










                answered 2 days ago









                VasyaVasya

                4,0681618




                4,0681618




















                    Max Power is a new contributor. Be nice, and check out our Code of Conduct.









                    draft saved

                    draft discarded


















                    Max Power is a new contributor. Be nice, and check out our Code of Conduct.












                    Max Power is a new contributor. Be nice, and check out our Code of Conduct.











                    Max Power is a new contributor. Be nice, and check out our Code of Conduct.














                    Thanks for contributing an answer to Mathematics Stack Exchange!


                    • Please be sure to answer the question. Provide details and share your research!

                    But avoid


                    • Asking for help, clarification, or responding to other answers.

                    • Making statements based on opinion; back them up with references or personal experience.

                    Use MathJax to format equations. MathJax reference.


                    To learn more, see our tips on writing great answers.




                    draft saved


                    draft discarded














                    StackExchange.ready(
                    function ()
                    StackExchange.openid.initPostLogin('.new-post-login', 'https%3a%2f%2fmath.stackexchange.com%2fquestions%2f3141677%2frelated-rates-my-answer-differs-from-the-book-misprint-or-me%23new-answer', 'question_page');

                    );

                    Post as a guest















                    Required, but never shown





















































                    Required, but never shown














                    Required, but never shown












                    Required, but never shown







                    Required, but never shown

































                    Required, but never shown














                    Required, but never shown












                    Required, but never shown







                    Required, but never shown







                    Popular posts from this blog

                    Lowndes Grove History Architecture References Navigation menu32°48′6″N 79°57′58″W / 32.80167°N 79.96611°W / 32.80167; -79.9661132°48′6″N 79°57′58″W / 32.80167°N 79.96611°W / 32.80167; -79.9661178002500"National Register Information System"Historic houses of South Carolina"Lowndes Grove""+32° 48' 6.00", −79° 57' 58.00""Lowndes Grove, Charleston County (260 St. Margaret St., Charleston)""Lowndes Grove"The Charleston ExpositionIt Happened in South Carolina"Lowndes Grove (House), Saint Margaret Street & Sixth Avenue, Charleston, Charleston County, SC(Photographs)"Plantations of the Carolina Low Countrye

                    random experiment with two different functions on unit interval Announcing the arrival of Valued Associate #679: Cesar Manara Planned maintenance scheduled April 23, 2019 at 00:00UTC (8:00pm US/Eastern)Random variable and probability space notionsRandom Walk with EdgesFinding functions where the increase over a random interval is Poisson distributedNumber of days until dayCan an observed event in fact be of zero probability?Unit random processmodels of coins and uniform distributionHow to get the number of successes given $n$ trials , probability $P$ and a random variable $X$Absorbing Markov chain in a computer. Is “almost every” turned into always convergence in computer executions?Stopped random walk is not uniformly integrable

                    How should I support this large drywall patch? Planned maintenance scheduled April 23, 2019 at 00:00UTC (8:00pm US/Eastern) Announcing the arrival of Valued Associate #679: Cesar Manara Unicorn Meta Zoo #1: Why another podcast?How do I cover large gaps in drywall?How do I keep drywall around a patch from crumbling?Can I glue a second layer of drywall?How to patch long strip on drywall?Large drywall patch: how to avoid bulging seams?Drywall Mesh Patch vs. Bulge? To remove or not to remove?How to fix this drywall job?Prep drywall before backsplashWhat's the best way to fix this horrible drywall patch job?Drywall patching using 3M Patch Plus Primer